Calculus: Early Transcendentals (2nd Edition)

Published by Pearson
ISBN 10: 0321947347
ISBN 13: 978-0-32194-734-5

Chapter 2 - Limits - 2.5 Limits at Infinity - 2.5 Exercises - Page 97: 52

Answer

$$\eqalign{ & a.{\text{ No horizontal asymptotes}} \cr & b.{\text{ No vertical asymptotes}} \cr} $$

Work Step by Step

$$\eqalign{ & f\left( x \right) = \frac{{{x^2} - 4x + 3}}{{x - 1}} \cr & \cr & {\text{Factor the numerator}} \cr & f\left( x \right) = \frac{{\left( {x - 3} \right)\left( {x - 1} \right)}}{{x - 1}} \cr & {\text{Cancel the common factors}} \cr & f\left( x \right) = x - 3, x\ne3 \cr & \cr & a.{\text{ Calculate }}\mathop {\lim }\limits_{x \to \infty } f\left( x \right){\text{ and }}\mathop {\lim }\limits_{x \to - \infty } f\left( x \right) \cr & \mathop {\lim }\limits_{x \to \infty } f\left( x \right)\,\,\, \Rightarrow \,\,\,\,\mathop {\lim }\limits_{x \to \infty } \left( {x - 3} \right) \cr & \mathop {\lim }\limits_{x \to \infty } \left( {x - 3} \right) = \infty \cr & \cr & \mathop {\lim }\limits_{x \to \infty } f\left( x \right)\,\,\, \Rightarrow \,\,\,\,\mathop {\lim }\limits_{x \to - \infty } \left( {x - 3} \right) \cr & \mathop {\lim }\limits_{x \to \infty } \left( {x - 3} \right) = - \infty \cr & \cr & {\text{The limits are not finite}},{\text{ so the graph of the function has no }} \cr & {\text{horizontal asymptotes}}{\text{.}} \cr & \cr & b.{\text{ Find the vertical asymptotes}} \cr & {\text{The function }}f\left( x \right) = x - 3, x\ne 3{\text{ is a straight line, so the graph of the }} \cr & {\text{function has no vertical asymptotes}}{\text{.}} \cr & \cr & a.{\text{ No horizontal asymptotes}} \cr & b.{\text{ No vertical asymptotes}} \cr} $$
Update this answer!

You can help us out by revising, improving and updating this answer.

Update this answer

After you claim an answer you’ll have 24 hours to send in a draft. An editor will review the submission and either publish your submission or provide feedback.